Support the author’s claim EXCEPT

This topic has expert replies
Master | Next Rank: 500 Posts
Posts: 292
Joined: Fri Jul 17, 2009 8:39 am
Thanked: 6 times
Followed by:1 members

Support the author’s claim EXCEPT

by pnk » Sun Oct 04, 2009 5:13 pm
Can any one answer this pls:

Source: Princeton Tests

Laying down new fiber-optic cables across America to replace old copper wire telephone cables will result in large increases in revenues because of improved capacity, even though the initial cost of the modernization program will be very high.

All of the following statements, if true, support the author's claim EXCEPT:

A) Scientists are currently working on software technology that would greatly increase the effective capacity of copper cable.

B)Communications experts predict that the demand for higher-capacity cables will continue to increase.

C) Systems using fiber-optic cables require less frequent maintenance than do those using copper cables.

D) New on-line computer services transmit an unprecedented amount of data and therefore require cables with increased transmission capacity.

E) Transmission systems that do not require cable will continue to be cost-prohibitive for many years to come.

OA is A

Master | Next Rank: 500 Posts
Posts: 239
Joined: Wed Feb 11, 2009 2:50 am

by delhiboy1979 » Sun Oct 04, 2009 11:07 pm
Well, looks quite straight forward

A) should be the correct answer, weakens the claim

B) strengthens

C) strengthens

D) strengthens
E) Out of scope.

Master | Next Rank: 500 Posts
Posts: 295
Joined: Tue Jul 15, 2008 10:07 am
Thanked: 4 times
GMAT Score:690

by vaibhav.iit2002 » Mon Oct 05, 2009 6:46 am
delhiboy1979 wrote:Well, looks quite straight forward

A) should be the correct answer, weakens the claim

B) strengthens

C) strengthens

D) strengthens
E) Out of scope.
I think A would have been correct, Scientists were working on fiber-optic cables.

C seems ok. How OA can be A?

Legendary Member
Posts: 882
Joined: Fri Feb 20, 2009 2:57 pm
Thanked: 15 times
Followed by:1 members
GMAT Score:690

by crackgmat007 » Mon Oct 05, 2009 11:31 am
delhiboy1979 wrote:Well, looks quite straight forward

A) should be the correct answer, weakens the claim

B) strengthens

C) strengthens

D) strengthens
E) Out of scope.
If the question states 'support the author's claim EXCEPT', we need to eliminate choices that support. One left can be Weaken or out of scope or neutral. I want to understand why E is not the answer. Anyone?

Legendary Member
Posts: 1161
Joined: Mon May 12, 2008 2:52 am
Location: Sydney
Thanked: 23 times
Followed by:1 members

by mehravikas » Mon Oct 05, 2009 12:26 pm
Can you explain your reason to eliminate 'E'.

How does 'A' supports the author's claim?
delhiboy1979 wrote:Well, looks quite straight forward

A) should be the correct answer, weakens the claim

B) strengthens

C) strengthens

D) strengthens
E) Out of scope.

Master | Next Rank: 500 Posts
Posts: 292
Joined: Fri Jul 17, 2009 8:39 am
Thanked: 6 times
Followed by:1 members

by pnk » Mon Oct 05, 2009 4:04 pm
OA is indeed A. But confusion remains between A & E.

'Support the claim EXCEPT' implies choices that does not support (it could weaken/neutral/out of scope) - Secondly, A talks about increase in effective capacity strengthening the claim, whereas E is out of scope....E should hv been the answer. Can we discuss why A, why not E

Legendary Member
Posts: 1161
Joined: Mon May 12, 2008 2:52 am
Location: Sydney
Thanked: 23 times
Followed by:1 members

by mehravikas » Mon Oct 05, 2009 4:09 pm
'A' should not be the answer...anyday 'E' is a better option than 'A'.
pnk wrote:OA is indeed A. But confusion remains between A & E.

'Support the claim EXCEPT' implies choices that does not support (it could weaken/neutral/out of scope) - Secondly, A talks about increase in effective capacity strengthening the claim, whereas E is out of scope....E should hv been the answer. Can we discuss why A, why not E

Master | Next Rank: 500 Posts
Posts: 239
Joined: Wed Feb 11, 2009 2:50 am

by delhiboy1979 » Tue Oct 06, 2009 1:42 am
Ok, will try in more detail here:

The claim is that laying down the new fibre optic cables will result in more revenues. lets see the options now:

A) Scientists are currently working on software technology that would greatly increase the effective capacity of copper cable.


Scientists are already working on technologies to improve the copper cable, the above claim then does not justify the initial cost of laying down the fibre cables. The only justification to the high costs is that copper cables are lousy compared with the fibre optic ones. Now, if there are people figuring out to make copper cables better then would nt you rather invest money there instead of on laying down new cables


The only reason to reject E is that it does not talk at all about transmission required by copper cables. It can be any media and not just copper cables. It is a neutral option in that it neither strengthens not weakens the claim.
[/i]

Legendary Member
Posts: 1161
Joined: Mon May 12, 2008 2:52 am
Location: Sydney
Thanked: 23 times
Followed by:1 members

by mehravikas » Tue Oct 06, 2009 3:16 am
The question is about which of the following does not support author's claim.

Agree with you that 'E' is totally irrelevant but is there is defined rule to always choose an option that does not support over an option that is out of scope or irrelevant

Master | Next Rank: 500 Posts
Posts: 239
Joined: Wed Feb 11, 2009 2:50 am

by delhiboy1979 » Tue Oct 06, 2009 3:27 am
Personally, I look for something that weakens the claim. If there is no such option, I would go for an option that is out of scope

Master | Next Rank: 500 Posts
Posts: 256
Joined: Mon Aug 10, 2009 6:31 pm
Thanked: 3 times

by gmatv09 » Tue Oct 06, 2009 2:21 pm
IMO A

Concl: Laying down fiber optic cable --> increased revenue
Reason for incr reven --> Incr capacity

A) Scientists are currently working on software technology that would greatly increase the effective capacity of copper cable.
if the capacity of copper wires incr then there wouldnt be demand for fo cables - Correct

B)Communications experts predict that the demand for higher-capacity cables will continue to increase.
FO cables have higher capacity - Incorrect

C) Systems using fiber-optic cables require less frequent maintenance than do those using copper cables.
FO cables require < maint - Incorrect

D) New on-line computer services transmit an unprecedented amount of data and therefore require cables with increased transmission capacity.
Incre demand for FO cables - Incorrect

E) Transmission systems that do not require cable will continue to be cost-prohibitive for many years to come.
Trans systs w/o cables too expensive - Incorrect

Master | Next Rank: 500 Posts
Posts: 292
Joined: Fri Jul 17, 2009 8:39 am
Thanked: 6 times
Followed by:1 members

by pnk » Tue Oct 06, 2009 9:41 pm
Thanks. My reading of A was wrong - missed effective copper cables. Considering copper cable A will surely not support the claim (here weaken the claim). Therefore A should be the answer.

Secondly, E is out of scope (talks about system that does not require cable).

However, question asks for "support the author claim EXCEPT" - if my understanding is correct, any option which can't strengthen the argument can be a right choice for this type of question. In this case, any choice that either weaken or neutral or out of scope could be the answer. (Is this understanding correct??? - pls share your thought. Or does this sort of question look for an answer that basically weaken the claim)

From this point of view, I do not find a logical reason to eliminate E. Could you clarify.[/b]

Legendary Member
Posts: 882
Joined: Fri Feb 20, 2009 2:57 pm
Thanked: 15 times
Followed by:1 members
GMAT Score:690

by crackgmat007 » Tue Oct 06, 2009 9:52 pm
pnk wrote:Thanks. My reading of A was wrong - missed effective copper cables. Considering copper cable A will surely not support the claim (here weaken the claim). Therefore A should be the answer.

Secondly, E is out of scope (talks about system that does not require cable).

However, question asks for "support the author claim EXCEPT" - if my understanding is correct, any option which can't strengthen the argument can be a right choice for this type of question. In this case, any choice that either weaken or neutral or out of scope could be the answer. (Is this understanding correct??? - pls share your thought. Or does this sort of question look for an answer that basically weaken the claim)

From this point of view, I do not find a logical reason to eliminate E. Could you clarify.[/b]
Yes, your understanding is correct. Support Except means, a choice that doesnt support. can be a weakener or out of scope etc. I think E may lend some support to the argument. A strengthner can support the argument 100% or 1%. All we need to figure out is whether the choice makes the argument more plausible.

Legendary Member
Posts: 1161
Joined: Mon May 12, 2008 2:52 am
Location: Sydney
Thanked: 23 times
Followed by:1 members

by mehravikas » Wed Oct 07, 2009 12:40 am
@@crackgmat007,

How is 'E' supporting the author's claim? Isn't it completely out of scope.

Vikas
crackgmat007 wrote:
pnk wrote:Thanks. My reading of A was wrong - missed effective copper cables. Considering copper cable A will surely not support the claim (here weaken the claim). Therefore A should be the answer.

Secondly, E is out of scope (talks about system that does not require cable).

However, question asks for "support the author claim EXCEPT" - if my understanding is correct, any option which can't strengthen the argument can be a right choice for this type of question. In this case, any choice that either weaken or neutral or out of scope could be the answer. (Is this understanding correct??? - pls share your thought. Or does this sort of question look for an answer that basically weaken the claim)

From this point of view, I do not find a logical reason to eliminate E. Could you clarify.[/b]
Yes, your understanding is correct. Support Except means, a choice that doesnt support. can be a weakener or out of scope etc. I think E may lend some support to the argument. A strengthner can support the argument 100% or 1%. All we need to figure out is whether the choice makes the argument more plausible.

User avatar
GMAT Instructor
Posts: 5
Joined: Tue Oct 06, 2009 10:44 am
Location: Tempe, AZ
Thanked: 4 times
GMAT Score:790

by ChrisHinkle » Wed Oct 07, 2009 3:47 pm
Answer choice E does support the conclusion by eliminating a reasonable objection. If it were the case that a new cable-free system were out to become feasible, then upgrading the cable system right now would not be a financially smart decision.

So A definitely best. With support except it is very important that all answers eliminated do provide some sort of support. But in tough strengthen questions be careful about eliminating for out of scope - ruling out a possible objection is a form of strengthening.
Chris Hinkle
GMAT Instructor, The Princeton Review